Introduction

Multivariate order statistics especially Bivariate order statistics have attracted the interest of several researchers, for example, see [1]. The distribution of bivariate order statistics can be easily obtained from the bivariate binomial distribution, which was first introduced by [2]. Considering a bivariate sample, David et al. [3] studied the distribution of the sample rank for a concomitant of an order statistic. Bairamove and Kemalbay [4] introduced new modifications of bivariate binomial distribution, which can be applied to derive the distribution of bivariate order statistics if a certain number of observations are within the given threshold set. Barakat [5] derived the exact explicit expression for the product moments (of any order) of bivariate order statistics from any arbitrary continuous bivariate distribution function (df). Bairamove and Kemalbay [6] used the derived jpdf by [5] to derive the joint distribution on new sample rank of bivariate order statistics. Moreover, Barakat [7] studied the limit behavior of the extreme order statistics arising from n two-dimensional independent and non-identically distributed random vectors. The class of limit dfs of multivariate order statistics from independent and identical random vectors with random sample size was fully characterized by [8].

Consider n two-dimensional independent random vectors \({{\underline W}_{j}}=(X_{j},Y_{j})\), j=1,2,...,n, with the respective distribution function (df) \(F_{j}(\underline w)=F_{j}(x,y)= P(X_{j}\leq x, Y_{j}\leq y), j=1,2,...,n \). Let X1:nX2:n≤...≤Xn:n and Y1:nY2:n≤...≤Yn:n be the order statistics of the X and Y samples, respectively. The main object of this work is to derive the jpdf of the random vector \(Z_{k,k^{\prime }:n}= (X_{n-k+1:n},Y_{n-k^{\prime }+1:n})\phantom {\dot {i}\!},\) where 1≤k, kn. Let \(G_{j}(\underline {w})=P({\underline {W}}_{j}>\underline {w})\) be the survival function of \(F_{j}(\underline {w}), j=1,2,...,n\) and let F1,j(.), F2,j(.), G1,j(.)=1−F1,j(.) and G2,j(.)=1−F2,j(.) the marginal dfs and the marginal survival functions of \(\Phi _{k,k':n}= P(Z_{k,k':n}\leq \underline {w}),\ F_{j}(\underline {w})\) and \(~G_{j}(\underline {w}), j=1,2,...,n,\) respectively. Furthermore, let \({F_{j}}^{1,.}=\frac {\partial F_{j}(\underline {w})}{\partial {x}}\) and \({F_{j}}^{.,1}=\frac {\partial F_{j}(\underline {w})}{\partial {y}}.\) Also, the jpdf of \((X_{n-k+1:n},Y_{n-k^{\prime }+1:n})\phantom {\dot {i}\!}\) is conveniently denoted by \(\phantom {\dot {i}\!}f_{k,k^{\prime }:n}(\underline {w}).\) Finally, the abbreviations min(a, b)=ab, and max(a, b)=ab will be adopted.

The jpdf of non-identical bivariate order statistics

The following theorem gives the exact formula of the jpdf of non-identical bivariate order statistics.

Theorem 1

The jpdf of non-identical bivariate order statistics is given by

$${{} \begin{aligned} f_{k,k':n}(\underline{w})\,=\,\sum_{\theta,\varphi=0}^{1}\sum_{r=r_{**}}^{r^{**}}\sum_{\rho_{\theta,\varphi,r}}\, \Pi_{j=1}^{\theta}{F}^{.,1}_{i_{j}}(\underline{w})\Pi_{j=\theta+1}^{1}(f_{2,i_{j}}(y)\,-\,{F}^{.,1}_{i_{j}}(\underline{w})) \Pi_{j=2}^{\varphi+1}{F}^{1,.}_{i_{j}}(\underline{w})\\ \times\Pi_{j=\varphi+2}^{2}(f_{1,i_{j}}(x)-{F}^{1,.}_{i_{j}}(\underline{w}))\Pi_{j=3}^{k-\theta-r+1} (F_{1,i_{j}}(x)-{F}_{i_{j}}(\underline{w}))\Pi_{j=k-\theta-r+2}^{k-\theta+1}F_{i_{j}}(\underline{w})\\ \times \Pi_{j=k-\theta+2}^{k+k'-\theta-\varphi-r}(F_{2,i_{j}}(y)-{F}_{i_{j}}(\underline{w})) \Pi_{j=k+k'-\theta-\varphi-r+1}^{n}G_{i_{j}}(\underline{w})+\sum_{r=0\vee(k+k'-n-1)}^{(k-1)\wedge(k'-1)}\sum_{\rho_{r}}f_{j}(\underline{w})\\ \Pi_{j=2}^{k-r}(F_{1,i_{j}}(x)-{F}_{i_{j}}(\underline{w})) \times\Pi_{j=k-r+1}^{k}F_{i_{j}}(\underline{w})\Pi_{j=k+1}^{k+k'-r}(F_{2,i_{j}}(y)-{F}_{i_{j}}(\underline{w}))\Pi_{j=k+k'-r+1}^{n}G_{i_{j}}(\underline{w}), \end{aligned}} $$

where \(r_{**}=0\vee (k+k'-\theta -\varphi -n), r^{**}= (k-\theta -1)\wedge (k'-\varphi -1),\ \sum _{\rho }\) denotes summation subject to the condition ρ, and \(\sum _{\rho _{\theta _{1},\theta _{2},\varphi _{1},\varphi _{2},\omega,r}}\) denotes the set of permutations of i1,...,in such that \(i_{j_{1}}<...< i_{j_{n}}.\)

Proof

A convenient expression of \(f_{k,k':n}(\underline {w})\) may derived by noting that the compound event E={x<Xk:n<x+δx, y<Yk:n<y+δy} may be realized as follows: r;φ1;s1;θ1;ω;θ2;s2;φ2 and t observations must fall respectively in the regions I1=(−,x]∩(−,y];I2=(x, x+δx]∩(−,y];I3=(x+δx,]∩(−,y];I4=(−,x]∩(y, y+δy];I5=(x, x+δx]∩(y, y+δy];I6=(x+δx,]∩(y, y+δy];I7=(−,x]∩(y+δy,);I8=(x, x+δx]∩(x+δx,);and I9=(x+δx,)∩(y+δy,) with the corresponding probability \(P_{ij}=P({\underline {W}}_{j}\in I_{i}), i=1,2,...,9\). Therefore, the joint density function \(f_{k,k':n}(\underline {w})\) of \(\phantom {\dot {i}\!}(X_{k:n},Y_{k^{\prime }:n})\) is the limit of \(\frac {P(E)}{\delta x\delta y}\) as δx,δy→0, where P(E) can be derived by noting that \(\theta _{1}+\theta _{2}+\omega =\varphi _{1}+\varphi _{2}+\omega =1;\ r+\theta _{1}+s_{2}=k-1;\ r+\varphi _{1}+s_{1}=k'-1;\ r,\theta _{1},s_{2},\varphi _{1},\omega,\theta _{2},s_{1},\varphi _{2},t \geq 0;\ P_{1j}=F_{j}(\underline {w}),P_{2j}=F_{j}^{1,.}(\underline {w})\delta x, P_{3j}=F_{2,j}(y)-F_{j}(x+\delta x,y), P_{4j}= F_{j}^{.,1}(\underline {w})\delta y, P_{5j\cong }F_{j}^{1,1}(\underline {w})\delta x\delta y=f_{j}(\underline {w})\delta x\delta y, P_{6j}\cong (f_{2,j}(y)-F_{j}^{.,1}(\underline {w}+\delta \underline {w}))\delta y,\) where \(f_{2,j}(y)=\frac {\partial F_{2,j}(y)}{\partial y},j=1, 2,...,n,~ \partial \underline {w}=(\delta x,\delta y), \underline {w}+\delta \underline {w}=(x+\delta x,y+\delta y),P_{7j}=F_{1,j}(x)-F_{j}(x,y+\delta y), P_{8j}=(f_{1,j}(x)-F_{j}^{1,.}(\underline {w}+\delta \underline {w}))\delta {x}, P_{9j}=1-F_{1,j}(x+\delta x)-F_{2,j}(y+\delta y)+F_{j}(\underline {w}).\) Thus, we get

$$ {{} \begin{aligned} f_{k,k':n}(\underline{w})\!&=\!\sum_{\theta_{1},\varphi_{1},\theta_{2},\varphi_{2}=0}^{1}\sum_{r=r_{*}}^{r^{*}} \sum_{\rho_{\theta_{1},\theta_{2},\varphi_{1},\varphi_{2},\omega,r}}\,\Pi_{j=1}^{\theta_{1}}P_{4i_{j}}\Pi_{\theta_{1}+1}^{\theta_{1}+\varphi_{1}}P_{2i_{j}} \Pi_{j=\theta_{1}+\varphi_{1}+1}^{\theta_{1}+\varphi_{1}+\theta_{2}}P_{6i_{j}} \Pi_{j=\theta_{1}+\varphi_{1}+\theta_{2}+1}^{\theta_{1}+\varphi_{1}+\theta_{2}+\varphi_{2}}P_{8i_{j}}\\ &\Pi_{j=\theta_{1}+\varphi_{1}+\theta_{2}+\varphi_{2}\,+\,1}^{\theta_{1}+\varphi_{1}+\theta_{2}+\varphi_{2}+\omega}P_{5i_{j}} \Pi_{j=\theta_{1}+\varphi_{1}+\theta_{2}+\varphi_{2}+\omega+1}^{\theta_{2}+\varphi_{1}+\theta_{2}+\omega+k-r\!-1}P_{7i_{j}} \Pi_{j=\theta_{2}+\varphi_{1}+\varphi_{2}+\omega+k\!-r}^{\varphi_{1}\,+\,\theta_{2}+\varphi_{2}+\omega+k-1}P_{1i_{j}} \Pi_{j=\varphi_{1}\,+\,\theta_{2}+\varphi_{2}+\omega\!+k}^{\theta_{2}\!+\varphi_{2}+\omega+k+k'\!-r\,-\,2}P_{3i_{j}}\\ &\Pi_{j=\theta_{2}+\varphi_{2}+\omega+k+k'-r-1}^{n}P_{9i_{j}}, \end{aligned}} $$
(1)

where \(r_{*}=0\vee (k+k'+\theta _{2}+\varphi _{2}+\omega -r-1-n), r^{*}= (k-\theta _{1}-1)\wedge (k'-\varphi _{1}-1),\sum _{\rho }\) denotes summation subject to the condition ρ, and \(\sum _{\rho _{\theta _{1},\theta _{2},\varphi _{1},\varphi _{2},\omega,r}}\) denotes the set of permutations of i1,...,in such that \(i_{j_{1}}<...< i_{j_{n}}\) for each product of the type \(\Pi _{j=j_{1}}^{j_{2}}\). Moreover, if j1>j2, then \(\Pi _{j=j_{1}}^{j_{2}}=1\). But (1) can be written in the following simpler form

$${{} \begin{aligned} P(E)=\sum_{\theta,\varphi=0}^{1}\sum_{r=r_{**}}^{r^{**}}\sum_{\rho_{\theta,\varphi,r}}\,\Pi_{j=1}^{\theta}P_{4i_{j}}\Pi_{j= \theta+1}^{1}P_{6i_{j}}\Pi_{j=2}^{\varphi+1}P_{2i_{j}} \Pi_{j=\varphi+2}^{2}P_{8i_{j}}\Pi_{j=3}^{k-\theta-r+1}P_{7i_{j}}\Pi_{j=k-\theta-r+2}^{k-\theta+1}P_{1i_{j}} \\ \Pi_{j=k-\theta+2}^{k+k'-\theta-\varphi-r}P_{3i_{j}} \Pi_{j=k+k'-\theta-\varphi-r+1}^{n}P_{9i_{j}}+\sum_{r=0\vee(k+k'-n-1)}^{(k-1)\wedge(k'-1)}\sum_{\rho_{r}}P_{5i_{3}}\Pi_{j=2}^{k-r}P_{7i_{j}} \Pi_{j=k-r+1}^{k}P_{1i_{j}} \Pi_{j=k+1}^{k+k'-r}P_{3i_{j}}\Pi_{j=k+k'-r}^{n}P_{9i_{j}}, \end{aligned}} $$

where r∗∗=0∨(k+kθφn),r∗∗=(kθ−1)∧(kφ−1). Therefore,

$$ {{} \begin{aligned} f_{k,k':n}(\underline{w})=\sum_{\theta,\varphi=0}^{1}\sum_{r=r_{**}}^{r^{**}}\sum_{\rho_{\theta,\varphi,r}}\,\Pi_{j=1}^{\theta}P_{4i_{j}} \Pi_{j=\theta+1}^{1}P_{6i_{j}}\Pi_{j=2}^{\varphi+1}P_{2i_{j}} \Pi_{j=\varphi+2}^{2}P_{8i_{j}}\Pi_{j=3}^{k-\theta-r+1}P_{7i_{j}}\\ \Pi_{j=k-\theta-r+2}^{k-\theta+1}P_{1i_{j}}\Pi_{j=k-\theta+2}^{k+k'-\theta-\varphi-r}P_{3i_{j}} \Pi_{j=k+k'-\theta-\varphi-r+1}^{n}P_{9i_{j}}+\sum_{r=0\vee(k+k'-n-1)}^{(k-1)\wedge(k'-1)}\sum_{\rho_{r}}P_{5i_{3}}\Pi_{j=2}^{k-r}P_{7i_{j}}\\ \Pi_{j=k-r+1}^{k}P_{1i_{j}}\Pi_{j=k+1}^{k+k'-r}P_{3i_{j}}\Pi_{j=k+k'-r}^{n}P_{9i_{j}}. \end{aligned}} $$
(2)

Thus, we get

$$ {{} \begin{aligned} f_{k,k':n}(\underline{w})=\sum_{\theta,\varphi=0}^{1}\sum_{r=r_{**}}^{r^{**}}\sum_{\rho_{\theta,\varphi,r}}\, \Pi_{j=1}^{\theta}{F}^{.,1}_{i_{j}}(\underline{w})\Pi_{j=\theta+1}^{1}(f_{2,i_{j}}(y)-{F}^{.,1}_{i_{j}}(\underline{w})) \Pi_{j=2}^{\varphi+1}{F}^{1,.}_{i_{j}}(\underline{w})\\ \Pi_{j=\varphi+2}^{2}(f_{1,i_{j}}(x)-{F}^{1,.}_{i_{j}}(\underline{w}))\Pi_{j=3}^{k-\theta-r+1} (F_{2,i_{j}}(x)-{F}_{i_{j}}(\underline{w}))\Pi_{j=k-\theta-r+2}^{k-\theta+1}F_{i_{j}}(\underline{w}) \Pi_{j=k-\theta+2}^{k+k'-\theta-\varphi-r}(F_{2,i_{j}}(y)-{F}_{i_{j}}(\underline{w}))\\ \Pi_{j=k+k'-\theta-\varphi-r+1}^{n}G_{i_{j}}(\underline{w})+\sum_{r=0\vee(k+k'-n-1)}^{(k-1)\wedge(k'-1)}\sum_{\rho_{r}}f_{i_{3}}(\underline{w}) \Pi_{j=2}^{k-r}(F_{1i_{j}}(x)-{F}_{i_{j}}(\underline{w}))\\ \Pi_{j=k-r+1}^{k}F_{i_{j}}(\underline{w})\Pi_{j=k+1}^{k+k'-r}(F_{2,i_{j}}(y)-{F}_{i_{j}}(\underline{w}))\Pi_{j=k+k'-r+1}^{n}G_{i_{j}}(\underline{w}). \end{aligned}} $$
(3)

Hence, the proof.

Relation (3) may be written in term of permanents (c.f [9]) as follows:

$$ {{} \begin{aligned} f_{k,k':n}(\underline{w})= \sum_{\theta,\varphi=0}^{1}\sum_{r=r_{**}}^{r^{**}}\frac{1}{(k-\theta-r-1)!r!(k'-\varphi-r-1)!(n-k-k'+\varphi+\theta+r-1)!}\\ \begin{array}{ccccccc}\text{Per} [\underline{U}^{.,1}_{1,1}&(\underline{U}^{1}_{.,1}\,-\,\underline{U}^{.,1}_{1,1})&\underline{U}^{1,.}_{1,1}& \left(\underline{U}^{1}_{1,.}\,-\,\underline{U}^{1,.}_{1,1}\right)& \left(\underline{U}_{1,.}\,-\,\underline{U}_{1,1}\right)&\underline{U}_{1,1}&(\underline{U}_{.,1}\,-\,\underline{U}_{1,1})~\\ ~~ {\theta}~ &~ { 1-\theta}~~&~ {\varphi} ~~&~ {1-\varphi}~~&~ {k-\theta-r-1}~&~ {r}&~ {k'-\varphi-r-1}\\ (1-\underline{U}_{1,.}-\underline{U}_{1,.}+\underline{U}_{1,1}){\vphantom{\underline{U}^{.,1}_{1,1}}}]\\~~ {n-k-k'+\theta+\varphi+r-1} \end{array}\\ +\sum_{r=r_{*}}^{r^{*}}\frac{1}{(k-r)!r!(k'-r)!(n-k-k'+r)!} ~{\renewcommand{\arraystretch}{0.6} \begin{array}{cccccccc}\text{Per} [\underline{U}^{1,1}_{1,1}~&~(\underline{U}_{1,.}-\underline{U}_{1,1})~&\underline{U}_{1,1}~&~ (\underline{U}_{.,1}-\underline{U}_{1,1})~&~ (1-\underline{U}_{1,.}-\underline{U}_{1,.}+\underline{U}_{1,1})]\\ ~ {1}~ &~ {k-r}~~&~ {r} ~~&~ {k'-r}~&~ {n-k-k'+r-1}\end{array}}, \end{aligned}} $$
(4)

where \(~\underline U_{1,.}=(F_{11}(x_{1})~~F_{12}(x_{1})~...~ F_{1n}(x_{1}))',\ \underline U_{.,1}=(F_{2,1}(x_{2})~~F_{2,2}(x_{2})~...~ F_{2,n}(x_{2}))',\ \underline U_{1,1}=(F_{1}(\underline x)~~F_{2}(\underline x)~...~ F_{n}(\underline x))'\) and \(\underline 1\) is the n×1 column vector of ones. Moreover, if \({\underline {a}}_{1}, {\underline {a}}_{2},... \) are column vectors, then

$$\begin{array}{cccc} \text{Per}[&{\underline{a}}_1~~&~~{\underline{a}}_2~~&~~...]\\ &~~{i_{1}}~~&~~{i_{2}}~~&~~... \end{array} $$

will denote the matrix obtained by taking i1 copies of \({\underline {a}}_{1},\ i_{2}\) copies of \({\underline {a}}_{2},\) and so on.

Finally, when k=k=1, in (3), we get

$$\begin{array}{*{20}l} f_{1,1:n}(\underline{w})=\sum_{\rho_{\theta,\varphi,r}}\, (f_{2,i_{1}}(y)-{F}^{.,1}_{i_{1}}(\underline{w})) (f_{1,i_{2}}(x)-{F}^{1,.}_{i_{2}}(\underline{w}))\Pi_{j=3}^{n} G_{i_{j}}(\underline{w})+\sum_{\rho_{r}}f_{i_{3}}(\underline{w})\\ (F_{2,i_{2}}(y)-{F}_{i_{2}}(\underline{w}))\Pi_{j=3}^{n}G_{i_{3}}(\underline{w}). \end{array} $$

Also, for k=k=n, we get

$$\begin{array}{*{20}l} {} f_{n,n:n}(\underline{w})=\sum_{\rho_{\theta,\varphi,r}}\, {F}^{.,1}_{i_{1}}(\underline{w}){F}^{1,.}_{i_{2}}(\underline{w})\Pi_{j=3}^{n}F_{i_{j}}(\underline{w})+\sum_{\rho_{r}}f_{i_{3}}(\underline{w}) \Pi_{j=2}^{n}F_{i_{j}}(\underline{w}))(F_{2,i_{n+1}}(y)-{F}_{i_{n+1}}(\underline{w})). \end{array} $$

Joint distribution of the new sample rank of X r:n and Y s:n

Consider n two-dimensional independent vectors \(\underline {W}_{j}=(X_{j},Y_{j}),j= 1,...,n,\) with the respective df \(F_{j}(\underline {W})\) and the jpdf\(F_{j}(\underline {W})\). Further assume that (Xn+1,Yn+1), (Xn+2,Yn+2),..., (Xn+m,Yn+m), (m≥1) is another random sample with absolutely continuous df \(G^{*}_{j}(x,y), j= 1,...,m\) and jpdf gj(x, y). We assume that the two samples (Xn+1,Yn+1),(Xn+2,Yn+2),...,(Xn+m, Yn+m), (m≥1) and (X1,Y1),(X2,Y2),...,(Xn,Yn) are independent.

For 1≤r, sn, m≥1, we define the random variables η1 and η2 as follows:

$$\begin{array}{*{20}l} \eta_{1}=\sum_{i=1}^{m}I_{(X_{r:n}-X_{n+i})} \end{array} $$

and

$$\begin{array}{*{20}l} \eta_{2}=\sum_{i=1}^{m}I_{(Y_{s:n}-Y_{n+i})}, \end{array} $$

where I(x)=1 if x>0 and I(x)=0 if x≤0 is an indicator function. The random variables η1 and η2 are referred to as exceedance statistics. Clearly η1 shows the total number of new X observations Xn+1,Xn+2,..., Xn+m which does not exceed a random threshold based on the rth order statistic Xr:n. Similarly, η2 is the number of new observations Yn+1,Yn+2,...,Yn+m which does not exceed Ys:n.

The random variable ζ1=η1+1 indicates the rank of Xr:n in the new sample Xn+1, Xn+2,..., Xn+m, and the random variable ζ2=η2+1 indicates the rank of Ys:n in the new sample Yn+1,Yn+2,...,Yn+m. We are interested in the joint probability mass function of random variables ζ1 and ζ2. We will need the following representation of the compound event P(ζ1=p,ζ2=q)=P(η1=p−1,η2=q−1).

Definition 1

Denote A={Xn+iXr:n},Ac={Xn+i>Xr:n}, B={Yn+iYs:n} and Bc={Yn+i>Ys:n}. Assume that in a fourfold sampling scheme, the outcome of the random experiment is one of the events A or Ac, and simultaneously one of B or Bc, where Ac is the complement of A.

In m independent repetitions of this experiment, if A appears together with B times, then A and Bc appear together p−1 times. Therefore, B appears together with Ac q−1 times and Bc mpq++2 times. This can be described as follows:

Clearly, the random variables η1 and η2 are the number of occurrences of the events A and B in m independent trials of the fourfold sampling scheme, respectively. By conditioning on Xr:n=x and Ys:n=y, the joint distribution of η1 and η2 can be obtained from bivariate binomial distribution considering the four sampling scheme with events A={Xn+ix}, B={Yn+iy}, and with respective probabilities

$$\begin{array}{*{20}l} P(AB)=P(X_{n+i}\leq x,Y_{n+i}\leq y),\\ P(AB^{c})=P(X_{n+i}\leq x,Y_{n+i}> y),\\ P(A^{c}B)=P(X_{n+i}> x,Y_{n+i}\leq y),\\ P(A^{c}B^{c})=P(X_{n+i}> x,Y_{n+i}> y). \end{array} $$

Now, we can state the following theorem.

Theorem 2

The joint probability mass function of ζ1 and ζ2, is given by

$${{} \begin{aligned} &P(\zeta_{1}=p, \zeta_{2}=q) = P(\eta_{1}= p-1,\eta_{2}=q-1)= \sum_{\ell=max (0, p+q-m-2)}^{min (p-1,q-1)}\int_{-\infty}^{\infty}\int_{-\infty}^{\infty}\\&\Pi_{j=1}^{\ell}G^{*}_{i_{j}}(x,y) \Pi_{j=\ell+1}^{p-1}[G^{*}_{1,i_{j}}(x)-G^{*}_{i_{j}}(x,y)]\Pi_{j=p}^{q-\ell-1+p}\left[G^{*}_{2,i_{j}}(y)-G^{*}_{i_{j}}(x,y)\right] \Pi_{j=q-\ell+p}^{m+2}\overline{G}^{*}_{1,i_{j}}(x) f_{k,k':n(\underline{w})} dxdy, \end{aligned}} $$

where, \(p,q= 1,...,m+1,\ f_{k,\acute {k}:n}(\underline {w})\) is defined in (3).

Proof

Consider the fourfold sampling scheme described in Definition (1). By conditioning with respect to Xr:n=x and Ys:n=y, we obtain

$$ {{} \begin{aligned} P(\zeta_{1}=p, \zeta_{2}=q) \equiv P(\eta_{1}= p-1,\eta_{2}=q-1)= P\left\{\sum_{i=1}^{m}I_{(X_{r:n}-X_{n+i})}=p-1,I_{(Y_{r:n}-Y_{n+i})}=q-1\right\}\\ =\int_{-\infty}^{\infty}\int_{-\infty}^{\infty}P\left\{\sum_{i=1}^{m}I_{(X_{r:n}-X_{n+i})}=p-1,I_{(Y_{s:n}-Y_{n+i})}=q-1| X_{r:n}=x, Y_{s:n}=y\right\}\\ \times P\{X_{r:n} = x, Y_{s:n} = y \} dxdy\\ =\int_{-\infty}^{\infty}\int_{-\infty}^{\infty}P\left\{\sum_{i=1}^{m}I_{(x-X_{n+i})}=p-1,I_{(y-Y_{n+i})}=q-1\right\}{dF}_{r,s:n}(x,y). \end{aligned}} $$
(5)

On the other hand,

$$ {{} \begin{aligned} P\left(\sum_{i=1}^{m}I_{(x-X_{n+i})}=p-1,I_{(y-Y_{n+i})}=q-1\right)=\sum_{\ell=max (0,p+q-m-2)}^{min(p-1,q-1)}\Pi_{j=1}^{\ell}P_{i_{j}}(AB)\Pi_{j=\ell+1}^{p-1}P_{i_{j}}(AB^{c})\\ \Pi_{j=p}^{q-\ell-2+p}P_{i_{j}}\Pi_{j=q-\ell-1+p}^{m}P_{i_{j}}. \end{aligned}} $$
(6)

Substituting (6) in (5), we get

$${{} \begin{aligned} P(\zeta_{1}=p, \zeta_{2}=q) = P(\eta_{1}= p-1,\eta_{2}=q-1)= \sum_{\ell=max (0, p+q-m-2)}^{min (p-1,q-1)}\int_{-\infty}^{\infty}\int_{-\infty}^{\infty}\Pi_{j=1}^{\ell}G^{*}_{i_{j}}(x,y)\\ \Pi_{j=\ell+1}^{p-1}[G^{*}_{1,i_{j}}(x)-G^{*}_{i_{j}}(x,y)]\Pi_{j=p}^{q-\ell-1+p}[G^{*}_{2,i_{j}}(y)-G^{*}_{i_{j}}(x,y)]\Pi_{j =q-\ell+p}^{m}\overline{G}^{*}_{1,i_{j}}(x) f_{k,k':n(\underline{w})} dxdy, \end{aligned}} $$

where p, q=1,...,m+1. This completes the proof.